LSAT and Law School Admissions Forum

Get expert LSAT preparation and law school admissions advice from PowerScore Test Preparation.

 Administrator
PowerScore Staff
  • PowerScore Staff
  • Posts: 8919
  • Joined: Feb 02, 2011
|
#59025
Please post your questions below!
 Andro10
  • Posts: 4
  • Joined: Jun 17, 2018
|
#59226
Hello, Could you please explain this one for me?
 Jon Denning
PowerScore Staff
  • PowerScore Staff
  • Posts: 904
  • Joined: Apr 11, 2011
|
#59450
Hey Andro - no problem! Let's see if we can sort this one out :)

To start with, we need to make sure we're clear on two things: (1) exactly what the conclusion is, if one is present; (2) the question type we're seeing. So we'll begin with the stimulus and any argumentation/reasoning there, and then discuss what we're asked to do!

The argument here is a bit convoluted—no surprise from question 21—but to sum it up: the Economist presents a belief in the last sentence, that total bank lending to companies is less than it was five years ago. This is based on a set of facts where banks can't charge large, financially strong companies interest rates that are as high as the ones the banks pay for that same money (the money they hope to lend), banks also won't lend to companies that are not financially strong, and finally banks' total lending to small and medium-sized companies is less than it was five years ago.

In fact, let's enumerate those three premise points to really highlight what this conclusion rests upon:

..... 1. Banks can't charge large, financially strong companies interest rates that are as high as the ones the
..... banks pay for that same money

..... 2. Banks won't lend to companies that are not financially strong

..... 3. Banks' total lending to small and medium-sized companies is less than it was five years ago

Now re-examine the exact conclusion: total bank lending to companies is less than it was five years ago. [emphasis mine]

When you consider that the conclusion is arguing about an amount—total lending—and compare that idea to the three premise points above, you should notice that only points 2 and 3 are also about lending volume! That is, in point 2 the banks don't lend to companies that are not financially strong, so that's zero lending at present (so certainly not more than five years ago). And in point 3 the banks are lending less to small and medium-sized companies than they were five years ago, so that's a lower amount now (a decrease, which aligns with the author's conclusion).

But what about point 1? All we're told there is that banks pay higher rates to borrow than they can charge to lend to large, financially strong companies...but we know nothing about lending volume from this! And that's the potential flaw in this argument: just because banks can only lend to large companies at lower rates than the bank pays, or might even take a financial loss in the process, doesn't mean they won't lend, and it certainly doesn't tell us how much lending-volume change has resulted! So with that unknown aspect, the conclusion as given is not yet valid/knowable.

The question itself is Justify the Conclusion, where we want an answer choice that would prove that banks are lending less to companies now than five years ago. And to prove that belief, we MUST address the glaring uncertainty about lending volume to large, financially strong companies (lending at a lower rate than that originally paid). Specifically, it would be helpful if we could say that banks now lend less under those circumstances than they did five years ago, or that banks don't lend at all in cases where the rate they pay is higher than the rate they can charge.

And that's precisely what answer choice (A) tells us: banks don't lend at rates that are lower than the rates they pay to borrow.

If that's true, then banks aren't lending to large, financially strong companies. Further, from points 2 and 3 listed above, banks aren't lending to any companies that aren't financially strong, and are lending less to small and medium-sized companies. So that's now zero lending volume for point 1. Zero volume for point 2. And lower volume for point 3. Considering that covers all company types (small, medium, and large), then cumulatively current lending volume must be lower now than it was five years ago, and the author is proven correct!

It's a tricky question because it hinges on spotting the element that's still uncertain (lending volume based on interest rates for large, financially strong companies) and then resolving it, but answer choice (A) does that beautifully.

I hope that helps!

Jon
 AM4747
  • Posts: 17
  • Joined: Oct 22, 2018
|
#61951
Hi Jon,

Thanks so much for this detailed explanation.

Just a follow up. What I construed from your discussion of volume is that in the absence of our correct answer choice A, we still have the hypothetical possibility of banks lending a huge amount of money with very low interest rates (i.e. significantly lower than the ones they have to pay) to a large financially strong company. That is, it could be the case that the banks, though decreasing the number of institutions to which they lend money, actually have an increase in the total dollars it lends.

To use a not-so-perfect analogy, I can lend 20 dollars to three people in one case, and fifty dollars to two people in another case. The conclusion in the stimulus does not necessarily depend on the number of institutions which receive loan money.

I hope my reasoning is correct
Thanks in advance
All best
 KSL
  • Posts: 12
  • Joined: Oct 13, 2018
|
#61983
Hello,

I am a bit confused. The stimulus says “currently the interest rates that banks pay to borrow are higher than the interest rates that they can receive for loans to large, financially strong companies”

Can we just completely disregard the truth of the stimulus because the answer choice justifies the conclusion? I feel like that is taking a step farther than just bringing in new information.

Thanks!
 Malila Robinson
PowerScore Staff
  • PowerScore Staff
  • Posts: 296
  • Joined: Feb 01, 2018
|
#62263
Hi KSL,
A Justify the Conclusion question is in the second family (The Help Family), so the truth that we are focusing on is in the answer choices. And specifically for a Justify question we are saying: which one of the answer choices, if it is true, would make the argument 100% correct. So no, we are not disregarding what is being said in the argument, nor are we focusing on the conclusion at the expense of the premises. What we need is another premise which, when added to the stimulus, makes the argument 100% true. In this case Answer A does that, but it doesn't harm any of the premises, instead it clarifies the first sentence. (Please check Jon's answer above for a detailed explanation of how it fixes the weakness in the argument.)
Hope that helps!
-Malila
 Lily123
  • Posts: 23
  • Joined: Apr 12, 2019
|
#65251
AM4747 wrote:in the absence of our correct answer choice A, we still have the hypothetical possibility of banks lending a huge amount of money with very low interest rates (i.e. significantly lower than the ones they have to pay) to a large financially strong company.
...
The conclusion in the stimulus does not necessarily depend on the number of institutions which receive loan money.
I think you have to pick how you want to define LENDING (number of institutions OR amount of money) & as long as you apply it consistently your result will be the same:
- If you're interpreting the premise about small & medium firms as a decrease in NUMBER of companies, then the conclusion must be about a decrease in the total NUMBER of companies

- If you're interpreting the premise about small & medium firms as a decrease in DOLLARS, then the conclusion should also be interpreted to be about DOLLARS.
The stimulus is much easier to understand if we don't equivocate around the meaning of "lending"

                                        5 Years Ago          Current
Small & medium               100                    50
Large                                      100                 100
Total                                        200                150

This example shows that it doesn't really matter how MUCH was lent to large companies. As long as the amount (dollars OR number of companies) is equal to or less than the amount it was 5 years ago, the conclusion will be true.

What (A) does is make the current amount to large companies = 0
AM4747 wrote:That is, it could be the case that the banks, though decreasing the number of institutions to which they lend money, actually have an increase in the total dollars it lends.
If we assign 2 different meanings to "lending", then even with (A), could it not still be the case that banks decreased the number of small & medium institutions AND increased total dollars lent to these companies? Sure! The reason is, there is no way to make the math work unless variables are referring to the same thing.

Hope that helps!
User avatar
 lavalsat
  • Posts: 13
  • Joined: Jan 26, 2021
|
#85932
For answer A to be correct, wouldn't one need to assume that "small" "medium" and "large" encompasses all of companies that banks can lend to? I see how that small/medium/large kind of does that- but not in a way where it would be watertight enough to be included in an LSAT answer choice. Why can't there exist companies that are "extra large" that received lots of lending? The small/medium/large designation really threw me off. I am missing something? Thanks
 Adam Tyson
PowerScore Staff
  • PowerScore Staff
  • Posts: 5153
  • Joined: Apr 14, 2011
|
#86533
If we are talking about shirt sizes, lavalsat, then "large" wouldn't include extra large or XXL, etc. But when we apply it to things like companies it's fair to say that "large" covers everything that is not medium or small. An extra large company is still a large company, even if it is a little extra!
User avatar
 novthe13th
  • Posts: 1
  • Joined: Oct 06, 2021
|
#91077
Can the negation test that we use for assumption questions also be extended to justify questions as well? I assumed this question was an assumption question and tried to use the negation test, but still got it wrong.

Get the most out of your LSAT Prep Plus subscription.

Analyze and track your performance with our Testing and Analytics Package.